The algebraic form of an with no solution is?

1. a = a
2. x = a
3. a = b


Which one is the answer?

Answers

Answer 1
a = b

In algebra, X normally correlated with a specific variable so if x = anything, (x = 4) that is a real equation. a = b just means a number equal to another number (3 = 4) which is not true.

Related Questions

Is Scrooge a static or dynamic character? Explain your answer with examples from the story. Please be sure you do not use your book; only give examples from what you remember. Write at least three sentences.

Answers

Answer:

do u need help?

...uhmm...ok i will help u.....i-if t-thts o-ok with y-you i-guess

Jessica has £600.
She gives 50% to her mother.
Jessica gives 10% to her brother.
She keeps the rest.
How much money does Jessica keep?

Answers

Answer:

she keeps 270

Step-by-step explanation:

have of 600 is 300 10 percent of 300 is 30

300-30 is 270

a^1/2 x a^3/2 divide (a^3)^4

Answers

Answer:

3x

Step-by-step explanation:

The cost of a pizza varies directly as the square of its radius. If a pizza with a 6 inch radius costs $8.00, how much should a pizza with an 11 inch radius cost? Round to the nearest hundredth.

Answers

Answer:

Step-by-step explanation:

14.7$

Candy is sold out in 4 pound bags. How many full

Answers

Full what exactly it is not specified what you need

what is the perimeter, in grid units, of a regular octagon that has one side with endpoints (-4, -3) and (-4, 8)

Answers

Answer:

88

Step-by-step explanation:

Let's find the length of one side. The perimeter will be 8 times that since it's a regular octagon.

[tex]d= \sqrt{(-4+(-4))^2+(8-(-3))^2} = \sqrt{0^2+(11)^2}= 11[/tex]

Perimeter has to be [tex]8\times 11 = 88[/tex]

Select all ratios equivalent to 3:5

Answers

6:10 , 9:15, 12:20 , and 15:25
3/5 6/10 9/15 12/20
15/25 18/30 21/35 24/40
27/45 30/50 33/55 36/60
39/65 42/70 45/75 48/80
51/85 54/90 57/95 60/100
63/105 66/110 69/115 72/120
75/125 78/130 81/135 84/140
87/145 90/150 93/155 96/160
99/165 102/170 105/175 108/180
111/185 114/190 117/195 120/200
123/205 126/210 129/215 132/220
135/225 138/230 141/235 144/240
147/245 150/250 153/255 156/260 159/265 162/270 165/275 168/280
171/285 174/290 177/295 180/300
183/305 186/310 189/315 192/320
195/325 198/330 201/335 204/340
207/345 210/350 213/355 216/360
219/365 222/370 225/375 228/380
231/385 234/390 237/395 240/400
243/405 246/410 249/415 252/420
255/425 258/430 261/435 264/440
267/445 270/450 273/455 276/460 279/465 282/470 285/475 288/480
291/485 294/490 297/495 300/500


Ratios can also be written in fraction form

25 points and brainliest for correct answer pls

Answers

A = lw

A = 9

9 = lw

9 = (x + 2)(x + 3)

9 = x² + 3x + 2x + 6

9 = x² + 5x + 6

x² + 5x = 3

Answer:

A = lw

A = 9

9 = lw

9 = (x + 2)(x + 3)

9 = x² + 3x + 2x + 6

9 = x² + 5x + 6

x² + 5x = 3

Step-by-step explanation:

Can someone please give me the (Answers) to this? ... please ...



I dentify the idlowing pairs. of angles.



1.<1 and <4

2.<1 and <5

3. <2 and <8

4.<4 and <5

5. <3 and <5

6.<3 and <6

7.<2 and <6

8.<1 and <8 /
/
<————-1——2—————>
3 / 4
5 / 6
<————————————>
7 / 8
/

Answers

                  1  / 2

<————---———————>

              3 / 4

            5 / 6

<————————————>

       7 / 8

ANGLE TYPES: Vertical, Adjacent, Linear, Complementary, Supplementary, Co-Interior, Alternate Interior, Alternate Exterior, Corresponding,

ANSWERS:

1. Vertical

2. Corresponding

3. Supplementary

4. Alternate Interior

5. Co-Interior

6. Alternate Interior

7. Corresponding

8. Alternate Exterior

Show all work

which of the following lines is parallel to 2y=3x-1?

a. y = 1/2x - 1
b. 2y = x - 3
c. 4y = 6x + 8
d. y = 3x + 4
e. 3y = 2x - 3

Answers

Answer:

c. 4y=6x+8

Step-by-step explanation:

For lines to be parallel, they must have the same slope. To solve this problem, we need to isolate the y on one side of the equation (putting the equation into slope intercept form).

The equation itself must be divided by 2 to do that. We end up with y=1.5x-0.5. We need to find another equation that has 1.5 as the slope.

b. y=0.5x-1.5

c. y=1.5x+2

d. y=3x+4

e. y=2/3x-1

As we can see, the only other equation with a slope of 1.5 is C. Therefore, C is our answer.

If car number plates have 3 English alphabet letters followed by 3 digits, how many of these start with ABC?​

Answers

Answer:

351520000

Step-by-step explanation:

351520000

Explanation:

There are 10 possible digits for the numbers (0,1,2,...,9), and 26 possible letters (A,B,C,.....,Z).

Since repetitions are allowed, we have that for each letter used, 26 still remain for the next choice, and for each digit used, 10 still remain for the next choice.

Hence, by the multiplication principle, the total different number of letter permutations is  

26

×

26

×

26

=

17576

and the total number of digit permutations is  

10

×

10

×

10

=

1000

But now since they can be in any order, we basically require in how many different ways can 3 be chosen from 6 which is the combination

6

C

3

=

6

!

(

6

3

)

!

3

!

=

20

So for each of the original permutations of numbers and letters, we must consider the 20 different combinations of positions they can be in.

Therefore by the multiplication principle, the total possible different number plates is

17576

×

1000

×

20

=

351520000

How do you solve the equation 4a+b=12

Answers

Rearrange the equation by subtracting what is to the right of the equal sign from both sides of the equation :

4*a+b-(12)=0

Such an equation is usually written y=mx+b ("y=mx+c" in the UK).

"y=mx+b" is the formula of a straight line drawn on Cartesian coordinate system in which "y" is the vertical axis and "x" the horizontal axis.

In this formula :

y tells us how far up the line goes

x tells us how far along

m is the Slope or Gradient i.e. how steep the line is

b is the Y-intercept i.e. where the line crosses the Y axis

The X and Y intercepts and the Slope are called the line properties. We shall now graph the line 4a+b-12 = 0 and calculate its properties

Notice that when a = 0 the value of b is 12/1 so this line "cuts" the b axis at b=12.00000

b-intercept = 12/1 = 12.00000

When b = 0 the value of a is 3/1 Our line therefore "cuts" the a axis at a= 3.00000

a-intercept = 12/4 = 3

Slope is defined as the change in b divided by the change in a. We note that for a=0, the value of b is 12.000 and for a=2.000, the value of b is 4.000. So, for a change of 2.000 in a (The change in a is sometimes referred to as "RUN") we get a change of 4.000 - 12.000 = -8.000 in b. (The change in b is sometimes referred to as "RISE" and the Slope is m = RISE / RUN)

Slope = -8.000/2.000 = -4.000

Geometric figure: Straight Line

Slope = -8.000/2.000 = -4.000 a-intercept = 12/4 = 3 b-intercept = 12/1 = 12.00000

Answer:

Step-by-step explanation:

Without having a numerical value for a (or b), you can't "solve this equation."

You can, however, solve it for either a or b, but not both.

For a:  Subtract b from both sides.  This results in 4a = 12 - b.  Dividing both sides by 4 isolates a:  

        12 - b

a = --------------

            4

For b:  Subtract 4a from both sides.  Then:   b = 12 - 4a

     

In 1903, the Wright brothers flew their first airplane. Sixty-six years later, a man walked on the moon. In which year did a man walk on the moon?
A. 1966
B. 1969
C. 1837
D. 1970

Answers

B. 1969

Step-by-step explanation:

Hope it helped you! :) (Please correct me if I'm wrong.)

what is 84 divided by -12

Answers

Answer:

-7

Step-by-step explanation:

Answer:

-7

Step-by-step expla

-7 times -12 is 84

5% of ______ is 10
_____ % of 100 is 12

Answers

Step-by-step explanation:

let the number represent X

5/100xx=10

5x/100=10

then you cross multiply

5x=100x10

5x=1000

divide both sides by 5

X=1000/5 =200

What is (-i)^3
A. -i
B. i
C. -1
D.1

Answers

Answer:

i

Step-by-step explanation:

=> -i³

=> (-1 . i )³

=> -1³ . i³

=> -1 [ i² . i ]

=> -1 [ (√-1)² . i ]

=> -1 [ -1 . i ]

=> i

Answer:

B: i

Step-by-step explanation:

Algebric explanation: as long as you remember power rules you should be good:

[tex](-i)^3 = (-1)^3\times i^3 = -1 \times i^2\times i= (-1)\times(-1)\times i=1i[/tex]

Geometric explanation: In the complex plane you can think "multiplying by i" as "rotate 1/4 counterclockwise". [tex](-1)^3[/tex] is a real number, and it's simply [tex]-1[/tex]. Rotate that ccw once, you're at [tex]-i[/tex]. Again, and you're at 1. third time (is really the charm!) you end up at [tex]i[/tex].

Polar explanation Courtesy of euler's identity.

[tex](-i)^3 = (e^{-\frac{\pi}2i})^3=e^{-\frac32\pi i}= cos (-\frac32\pi)+isin(-\frac32\pi) = 0+i(1)= i[/tex]

For what values of a and m does f(x) have a horizontal asymptote at y = 2 and a vertical asymptote at x = 1? f (x) = StartFraction 2 x Superscript m Baseline Over x a EndFraction a = –1, m = 0 a = 1, m = 0 a = –1, m = 1 a = 1, m = 1.

Answers

Answer:

C. a= -1, m=1

Step-by-step explanation:

hope i helped, have a great day :)

For the function f(x) = [tex]\frac{2x^m}{ x + a}[/tex], if the horizontal asymptote is at y = 2 and vertical asymptote is at x = 1, then a = -1 and m = 1.

What are Asymptotes of a Function?

There are asymptotes like horizontal, vertical and oblique asymptotes.

Horizontal asymptote is the value of the function f(x) as x goes to ±∞, as long as this value is not infinity.

Vertical asymptotes are the values of x which are not in the domain of the function f(x). For a fractional expression it is found by letting the denominator equals zero.

The given function is,

f(x) =  [tex]\frac{2x^m}{ x + a}[/tex]

Horizontal asymptote of this function is at y = 2.

lim x → ∞ f(x) = 2

lim x → ∞  [tex]\frac{2x^m}{ x + a}[/tex] = 2

Eliminate 'a' in the denominator, because as x tends to infinity a + x also tends to infinity.

lim x → ∞  [tex]\frac{2x^m}{ x }[/tex] = 2

For the left hand side to become 2, [tex]x^m[/tex] in the numerator and x in the denominator should get cancelled.

⇒ [tex]x^m[/tex] = [tex]x[/tex]

⇒ m = 1

Vertical asymptote of the function is at x = 1.

Let x + a = 0

⇒ 1 + a = 0

⇒ a = -1

Hence we get the values a = -1 and m = 1 for the function f(x) =  [tex]\frac{2x^m}{ x + a}[/tex] to have the horizontal asymptote at y = 2 and vertical asymptote at x = 1.

To learn more on Asymptotes, click :

https://brainly.com/question/4084552

#SPJ5

Find a trinomial for the area of the rectangular rug shown on the right whose sides are x+2 feet and 3x-1 feet.

Answers

The trinomial for the rectangular rug is 3x² + 5x - 2

A polynomial is an expression consisting of the operations of addition, subtraction, multiplication of variables.

A trinomial is a polynomial with three terms.

The area of a rectangle = length * width

Area of rectangle = (x + 2) * (3x - 1) = 3x² + 6x - x - 2

Area of rectangle = 3x² + 5x - 2

The trinomial for the rectangular rug is 3x² + 5x - 2

Find out more at: https://brainly.com/question/18563587

in the difference in altitude between 2,993 feet on a mountain peak and −428 feet on a valley floor

what is the difference in altitude per feet

Answers

Is it 0 percent in feet

I suck at math help ill give brainliest ​

Answers

Answer:

3

Step-by-step explanation:

5x + 40 = 55

5x = 55 - 40

5x = 15

x = 15 / 5

x = 3

F(x)=-x^2-12 to find (-3)

Answers

Answer:

[tex]{ \boxed{ \mathfrak{ \: answer : \: { \rm{f( - 3) = - 21}} }}}[/tex]

Step-by-step explanation:

[tex]{ \rm{f(x) = - {x}^{2} - 12 }}[/tex]

• When x is -3:

[tex]{ \rm{f( - 3) = - {( - 3)}^{2} - 12}} \\ \\ { \rm{f( - 3) = - 9 - 12}} \\ \\ { \rm{f( - 3) = - 21}}[/tex]

Find the length of side x in simplest radical form with a rational denominator.

Answers

Answer:

x =  [tex]3\sqrt{2}[/tex]

Step-by-step explanation:

This is an isosceles triangle, so the missing side is also 3

x = [tex]\sqrt{3^2 + 3^2}[/tex]

  = [tex]\sqrt{9+9}[/tex]

  = [tex]\sqrt{18}[/tex]

Simplify [tex]\sqrt{18}[/tex]:

18 = 2 x 3 x 3

[tex]\sqrt{18}[/tex] = [tex]\sqrt{2 * 3 * 3}[/tex]

[tex]\sqrt{18}[/tex] = [tex]\sqrt{2 * 3^2}[/tex]

[tex]\sqrt{18}[/tex] = [tex]\sqrt{2} * \sqrt{3^2}[/tex]

[tex]\sqrt{18}[/tex] = [tex]3\sqrt{2}[/tex]

-Chetan K

Sean bought 1.8 pounds of gummy bears and 0.6 pounds of jelly beans and paid $10.26. He went back to the store the following week and bought 1.2 pounds of gummy bears and 1.5 pounds of jelly beans and paid $15.09. What is the price per pound of each type of candy?

Answers

Answer:

Gummy bears $3.20/lbs

Jelly beans $7.50/lbs

Step-by-step explanation:

For this we will substitute gummy bear price for "x" and jelly bean price for "y" which creates this system (also assuming there is no sales tax involved):

1.8x + 0.6y = 10.26

1.2x + 1.5y = 15.09

Then once solved, answers come out to be (x,y) = (16/5,15/2) which is (3.2,7.5)

what is 15/4 as a whole number i've never been good at this and could us help

Answers

Answer:

3

Step-by-step explanation:

15/4=3 with a remainder of 3 . You take the 3 from your answer and it becomes the whole number.

What is the soultion to the following system of equations
{x+4y=-22
{3x-7y=29

Answers

Answer:

y= -1/4x - 11/2

y= 3/7x - 29/7

PLEASE HELP I WILL GIVE BRAINLIEST

Answers

Answer:

N

Step-by-step explanation:

Answer:

n

Step-by-step explanation:

3m^2 + 5n; if m = -4 and n = -13

Answers

Answer:

3m^2 + 5n = 3

Step-by-step explanation:

3(16) + 5(-13) =

48 - 45 =

3

Write the phrase as an expression. Then evaluate when x=5 .

the sum of a number x and 4, all divided by 3

Expression:

When x=5, the value of the expression is :

Answers

Answer:

[tex]\frac{x+4}{3}[/tex] ; 3

Step-by-step explanation:

In this question, we're being asked to write an equation using the word format. Let's find key words in the statement.

The sum of a number x and 4, all divided by 3.

Looking at the wording, we can conclude that we are adding x and 4, and dividing that by 3. So here's how the phrase would look as an expression.

[tex]\frac{1}{3}[/tex] (x+4) or [tex]\frac{x+4}{3}[/tex]

Now, there's another part to this problem. We must evaluate the value of the expression if x is given to be 5. So let's plug it into our equations.

(5)+4

   3

    [tex]\frac{9}{3}[/tex]

9/3 gives you 3, so your answer would be 3.

Let me know if you see any errors or have any further questions, otherwise, I hope this helped! :)

-
How many solutions does the equation 2 + 6(x - 4) = 3x - 18 + 3x have?

A)0
B)1
C)2
D) Infinite

Answers

Answer:

C because it's a quadratic equation and it can be solved by factorization and completing the square method and also general formula

PLEASE HELP QUICK!!

what is the slope of this line?

Answers

I think it might be D

Positive 3. Take a point (1,2), (2,5) and take the formula y2-y1/ x2-x1. 3/1 is slope.
Other Questions
Which of the following describes the Bolsheviks' "war communism"?A terror campaign implemented by the ChekaO A policy designed to marshal all of Russia's economic resourcesThe application of the total-war concept to a civil conflictO The seizure and redistribution of land among the Russian peasants A large serving of soup from a take-out restaurant is 4/5 liter. The restaurant has 3 liters of soup. The diagram models the number of servings of soup.How many servings of soup does the restaurant have? Nouns Adjectives 1. amazingbeauty 2. terror 3. 4. proud bravery 5. 6. excited satisfaction 7. what does lol mean?! What is the y-intercept? 48 mouse traps/6 people = X mouse traps/1 person What is an advantage of being multicellular?-having lower energy requirements-producing offspring that mature quickly -having a high tolerance for extreme conditions-being more adaptive to the surroundings which of the following sentence ls use active voice Colin and Brian were playing darts. Colin scored 36. Brian scored 35 more than Colin. What was their combined score? Which of the following statements best compares reproduction in viruses with reproduction in single-celled protists? no links please A car travels the distance between logan airport and the town of framingham in 20 min. The logan express bus travels this distance in 30 min. After how long do the car and bus meet if they leave at the same time, and one travels from framingham to the airport while the other travels from the airport to framingham?. What does 6523 65 equal? Round 16.4829 to the nearest hundredth Estimate g(14). (Use the midpoint to get the most precise estimate.)I know how to apply the midpoint theorem, but I don't know where I should start my intervals. NEED HELP PLEASE!! 25 points to who gives correct answer!! Spanish Homework Dialogo con verbos en futuro Integrating 160 different nationalities into onecorporation, such as The Emirates Group, haschallenges and opportunities. What challengesdo you see? What opportunities come from thisdiverse workforce? look at picture, need to know net force, whether they are balanced or unbalanced and whether they are accelerated or not accelerated! The man in the well passage how do the children respond to the mans initial cry for help Help ASAP!!!!!A. 1/4B. 4C. - 1/4D. 4